Ask Your Question

Revision history [back]

click to hide/show revision 1
initial version

Evaluating values of the Weierstrass $\wp$-function

I would like to know how can we evaluate the Weierstrass $\wp$-functions. That is, I would like to find $\wp(\theta,\omega,i\omega)$ for some $\theta,\omega\in\mathbb{R}$.

I'm only able to find a function which outputs the Laurent series of the Weierstrass $\wp$-function when an elliptic curve has been entered. Should I evaluate that laurent series?